Quanten.de Diskussionsforum  

Zur?ck   Quanten.de Diskussionsforum > Quantenmechanik, Relativitätstheorie und der ganze Rest.

Hinweise

Quantenmechanik, Relativitätstheorie und der ganze Rest. Wenn Sie Themen diskutieren wollen, die mehr als Schulkenntnisse voraussetzen, sind Sie hier richtig. Keine Angst, ein Physikstudium ist nicht Voraussetzung, aber man sollte sich schon eingehender mit Physik beschäftigt haben.

Antwort
 
Themen-Optionen Ansicht
  #1  
Alt 14.01.12, 16:10
baggi baggi ist offline
Newbie
 
Registriert seit: 14.01.2012
Beitr?ge: 6
Standard Winkelanteil eines Orbitals

Hallo alle miteinander,
ich mussjetz einfach mal nachfragen, weil ich bei meiner Problematik nicht vorankomme. Und zwar habe ich folgende Aufgabenstellung:
"Skizzieren Sie den Winkelanteil eines Orbitals, dessen Wellenanteil mit
Ψ(θ,φ)=(5/8*π)^0,5 *cosθ *sinφ = (5/8*π)^0,5 * x/r gegeben ist!"

In emeinen Aufzeichnungen steht, dass der Winkelanteile nochmals aufgeteilt (separiert) sind: Ψ(θ,φ) = Θ(θ) *Η (φ)

mit: Η(φ) = (2π)^0,5 * exp (imφ)
Θ(θ) = N * "legendre-Polynom" kurz LP

aber durch das LP entstehen nur Faktoren mit dem Winkel θ.
Wie komme ich also von exp (imφ) auf das sinφ ?

Danke schonmal für die Hilfe^^
Mit Zitat antworten
  #2  
Alt 14.01.12, 17:32
Hawkwind Hawkwind ist offline
Singularität
 
Registriert seit: 22.07.2010
Ort: Rabenstein, Niederösterreich
Beitr?ge: 3.063
Standard AW: Winkelanteil eines Orbitals

Zitat:
Zitat von baggi Beitrag anzeigen
Hallo alle miteinander,
ich mussjetz einfach mal nachfragen, weil ich bei meiner Problematik nicht vorankomme. Und zwar habe ich folgende Aufgabenstellung:
"Skizzieren Sie den Winkelanteil eines Orbitals, dessen Wellenanteil mit
Ψ(θ,φ)=(5/8*π)^0,5 *cosθ *sinφ = (5/8*π)^0,5 * x/r gegeben ist!"

In emeinen Aufzeichnungen steht, dass der Winkelanteile nochmals aufgeteilt (separiert) sind: Ψ(θ,φ) = Θ(θ) *Η (φ)

mit: Η(φ) = (2π)^0,5 * exp (imφ)
Θ(θ) = N * "legendre-Polynom" kurz LP

aber durch das LP entstehen nur Faktoren mit dem Winkel θ.
Wie komme ich also von exp (imφ) auf das sinφ ?

Danke schonmal für die Hilfe^^
Da assoziiere ich die Identität des Hr. Euler:

Mit Zitat antworten
  #3  
Alt 14.01.12, 17:38
Benutzerbild von Marco Polo
Marco Polo Marco Polo ist offline
Moderator
 
Registriert seit: 01.05.2007
Beitr?ge: 4.998
Standard AW: Winkelanteil eines Orbitals

sin(x)=(e^ix-e^-ix)/2i

Gruss, MP
Mit Zitat antworten
  #4  
Alt 14.01.12, 17:46
Benutzerbild von Bauhof
Bauhof Bauhof ist offline
Singularität
 
Registriert seit: 07.12.2008
Ort: Nürnberg
Beitr?ge: 2.105
Standard AW: Winkelanteil eines Orbitals

Zitat:
Zitat von baggi Beitrag anzeigen
Wie komme ich also von exp (imφ) auf das sinφ ?
Hallo baggi,

was bedeutet m?

Vielleicht hilft das weiter:

sin(x) = [exp(ix) ─ exp(─ix)] / (2i)

M.f.G. Eugen Bauhof
__________________
Ach der Einstein, der schwänzte immer die Vorlesungen –
ihm hatte ich das gar nicht zugetraut!

Hermann Minkowski
Mit Zitat antworten
  #5  
Alt 14.01.12, 17:56
Benutzerbild von Marco Polo
Marco Polo Marco Polo ist offline
Moderator
 
Registriert seit: 01.05.2007
Beitr?ge: 4.998
Standard AW: Winkelanteil eines Orbitals

Hallo Eugen,

Zitat:
Zitat von Bauhof Beitrag anzeigen
was bedeutet m?
das "im" steht für Imaginärteil.

Gruss. MP
Mit Zitat antworten
  #6  
Alt 14.01.12, 18:09
baggi baggi ist offline
Newbie
 
Registriert seit: 14.01.2012
Beitr?ge: 6
Standard AW: Winkelanteil eines Orbitals

Hallo, also danke schonmal für die Einfälle. Aber mit den angegebenen Lösungsansätzen habe ich das schon probiert. Damit komme ich nicht wirklich voran. Habt ihr noch andere Einfälle?

m ist die Magnetquantenzahl. Man betrachtet ja hier ein Orbital mit bestimmten Quantenzahlen die mit n, l, m und s gekennzeichnet sind.

Die Thetafunktion lautet wie folgt (ich weiß ehrlich gesagt nicht, wie man hier die Formeln korrekt darstellen kann):

Θ(θ) = sqrt {(2l+1)/2 * (l - |m|!)/(l+|m|!)} P "links oben steht |m|" "links unten steht l" (cos θ) also:
Θ(θ) = Normierungsfaktor* Legendre-Polynom

für m = 0 würde ja der Exponentailanteil aber auch der phi Anteil wegfallen.
mit dem Legendre-Polynom erhalte ich nur trigonometrische Funktionen von theta.
Mit Zitat antworten
  #7  
Alt 14.01.12, 18:09
Hawkwind Hawkwind ist offline
Singularität
 
Registriert seit: 22.07.2010
Ort: Rabenstein, Niederösterreich
Beitr?ge: 3.063
Standard AW: Winkelanteil eines Orbitals

Zitat:
Zitat von Marco Polo Beitrag anzeigen
Hallo Eugen,



das "im" steht für Imaginärteil.

Gruss. MP
Endlich mal einen aus Herne-West erwischt!
Da irrst du, MP:

i = imaginäre Einheit
m = "magnetische Quantenzahl"

Der beschriebene Winkelanteil sieht nach Kugelflächenfunktion aus ("spherical harmonics"). So was kriegt man, wenn man die Schrödinger-Gleichung fürs H-Atom löst (wenn ich mich recht entsinne)
Mit Zitat antworten
  #8  
Alt 14.01.12, 18:11
Hawkwind Hawkwind ist offline
Singularität
 
Registriert seit: 22.07.2010
Ort: Rabenstein, Niederösterreich
Beitr?ge: 3.063
Standard AW: Winkelanteil eines Orbitals

Zitat:
Zitat von Marco Polo Beitrag anzeigen
sin(x)=(e^ix-e^-ix)/2i

Gruss, MP
Da hast du ausnahmsweise mal recht.
Mit Zitat antworten
  #9  
Alt 14.01.12, 18:14
Benutzerbild von Marco Polo
Marco Polo Marco Polo ist offline
Moderator
 
Registriert seit: 01.05.2007
Beitr?ge: 4.998
Standard AW: Winkelanteil eines Orbitals

Zitat:
Zitat von Hawkwind Beitrag anzeigen
Endlich mal einen aus Herne-West erwischt!
Da irrst du, MP:

i = imaginäre Einheit
m = "magnetische Quantenzahl"

Der beschriebene Winkelanteil sieht nach Kugelflächenfunktion aus ("spherical harmonics"). So was kriegt man, wenn man die Schrödinger-Gleichung fürs H-Atom löst (wenn ich mich recht entsinne)
Huch. Hast Recht. Na sowas...

Gruss, nach Lüdenscheid-Nord
Mit Zitat antworten
  #10  
Alt 14.01.12, 18:17
baggi baggi ist offline
Newbie
 
Registriert seit: 14.01.2012
Beitr?ge: 6
Standard AW: Winkelanteil eines Orbitals

Zitat:
Zitat von Hawkwind Beitrag anzeigen
Der beschriebene Winkelanteil sieht nach Kugelflächenfunktion aus ("spherical harmonics"). So was kriegt man, wenn man die Schrödinger-Gleichung fürs H-Atom löst (wenn ich mich recht entsinne)
Ja die Seite habe ich auch schon angeschaut und da fand ich das hier ganz gut:

cosφsinθ = "Real" (exp (iφ) sin θ)
aber da weiß ich nicht wie ich das weiter rechnerisch bzw darstellerisch verarbeiten kann.

BTW: ich habe die Wellenfunktionen bezüglich φ und θ (also bezüglich der Wellenanteile) für l bis 2 und jeweils m = -l ..0..+l schon da aber keines davon entspricht dem, was in der Aufgabe verlangt ist. Und das sind auch nicht die selben wie auf der Wikipedia-Seite. Darf man an der Unfehlbarkeit des Professors zweifeln?^^

...Ich hoffe ich mache es nicht zu kompliziert

Ge?ndert von baggi (14.01.12 um 18:26 Uhr)
Mit Zitat antworten
Antwort

Lesezeichen

Themen-Optionen
Ansicht

Forumregeln
Es ist Ihnen nicht erlaubt, neue Themen zu verfassen.
Es ist Ihnen nicht erlaubt, auf Beitr?ge zu antworten.
Es ist Ihnen nicht erlaubt, Anh?nge hochzuladen.
Es ist Ihnen nicht erlaubt, Ihre Beitr?ge zu bearbeiten.

BB-Code ist an.
Smileys sind an.
[IMG] Code ist an.
HTML-Code ist aus.

Gehe zu


Alle Zeitangaben in WEZ +1. Es ist jetzt 17:19 Uhr.


Powered by vBulletin® Version 3.8.8 (Deutsch)
Copyright ©2000 - 2024, vBulletin Solutions, Inc.
ScienceUp - Dr. Günter Sturm